The perimeter of triangle XYZ is 24 units.
What is the area of triangle XYZ? Round to the nearest
tenth of a square unit.
Trigonometric area formula: Area=1/2absin(C)
o 14.7
square units
14.9 square units
15.0 square units
15.3 square units

Answers

Answer 1

To the nearest tenth, the area of triangle XYZ is approximately 14.7 units².

What is a triangle?

A triangle is a 3-sided polygon that is occasionally (though not frequently) referred to as the trigon.

There are three sides and three angles in every triangle, some of which may be the same.

Triangles can be divided into three groups based on the lengths of their sides, and these groups are as follows: Scalene, Isosceles, and Equilateral.

So, the area of the triangle is:

We know that:

<YXZ = 102°

Length of XY = z = 3

Length of YZ = x = 11

Length of XZ = y = ?

The perimeter of the triangle:

x+y+z = 24 units

Length XZ = y:
11+y+3 = 24

14+y = 24

y = 24 - 14

Therefore, y = 0.

Area of triangle XYZ:

The formula for trigonometric area is area of XYZ = 12(yz)sin X.

Putting values of x, y, and z as follows:

Area of ∆ XYZ = ½×10×3×sin 102

= ½×30×0.9782

= 15 × 0.9782

= 14.673

Therefore, to the nearest tenth, the area of triangle XYZ is approximately 14.7 units².

Know more about triangles here:

https://brainly.com/question/28889256

#SPJ1

Correct question:

What is the area of triangle XYZ? Round to the nearest tenth of a square unit. Trigonometric area formula: Area 14.7 square units 14.9 square units 15.0 square units 15.3 square units

The Perimeter Of Triangle XYZ Is 24 Units.What Is The Area Of Triangle XYZ? Round To The Nearesttenth

Related Questions

Calculate the quotient below and give your answer in scientific notation.0.000655 x 10-2= ?

Answers

[tex]\frac{0.00065}{5\cdot10^{-2}}[/tex]

3^9/3^6= answer in exponential form

Answers

To express this fraction in exponential form we have to remember the following property:

[tex]\frac{a^m^{}}{a^n}=a^{m-n}[/tex]

Applying it to our problem we have:

[tex]\begin{gathered} \frac{3^9}{3^6}=3^{9-6} \\ =3^3 \end{gathered}[/tex]

So the exponential form of our fraction is

[tex]3^3[/tex]

The formula for the volume of a rectangular prism is found by multiplying the width, length, and height of the prism. In other words, V = lwh. Solve the formula for the width, w.

Answers

The formula for the volume of a rectangular prism is

[tex]V=l\cdot h\cdot w[/tex]

You need to write the formula for w, note that the width is being miltiplied by "lh"

to cancel this multiplication you have to divide it by "lh" and to keep the equality valid, what is done to one side of the expression must be done to the other, so divide V by "lh" too

[tex]w=\frac{V}{lh}[/tex]

Please do this fast and quick I need to sleep

Answers

Answer: [tex]\begin{gathered} Quadratic\text{ equation: }4.9t^2\text{ - 9.8t - 73.5} \\ The\text{ time it will take the cannonball to reach sea level is 5 seconds} \end{gathered}[/tex]

Explanation:

Given:

distance from sea level to top of hill = initial heeight = 73.5 meters

velocity = 9.8 m/s

[tex]\begin{gathered} For\text{ vertical movement:} \\ Final\text{ height = acceleration\lparen t}^2)\text{ + velocity\lparen t\rparen+ initial height} \\ Since\text{ it is reaching sea level, final height = 0} \\ acceleration\text{ = -9.8 m/s}^2 \\ \\ 0\text{ = -}\frac{1}{2}(9.8)t^2\text{ + 9.8t + 73.5m} \end{gathered}[/tex][tex]\begin{gathered} 0\text{ = -4.9t}^2\text{ + 9.8t + 73.5} \\ 4.9t^2\text{ - 9.8t - 73.5 = 0 \lparen quadratic equation\rparen} \\ \\ \text{Using formula method to find the value of t:} \\ t\text{ = }\frac{-b\pm\sqrt[]{b^2-4ac}}{2a} \\ t\text{ = }\frac{-(-9.8)\pm\sqrt{(-9.8)^2-4(4.9)(-73.5)}}{2(4.9)} \\ \text{ t = }\frac{-(-9.8)\pm\sqrt{1536.64}}{9.8} \\ t\text{ = }\frac{9.8\pm39.2}{9.8} \end{gathered}[/tex][tex]\begin{gathered} t\text{ = }\frac{9.8+39.2}{9.8}\text{ ot }\frac{9.8\text{ - 39.2}}{9.8} \\ \\ t\text{ = 5 or -3} \end{gathered}[/tex]

Since we can't have t to be negative, t = 5

The cannonball will reach the sea level at 5 seconds

Please see attachment for question.Fill in the table and then graph the function

Answers

ANSWER

EXPLANATION

First, we have to fill in the table. To do so, we will plug the x-values into the function to find the corresponding value of y,

[tex]\begin{cases}y=-3\cdot3^{-3}=-\frac{3}{3^3}=-\frac{3}{27}=-\frac{1}{9} \\ \\ y=-3\cdot3^{-2}=-\frac{3}{3^2^{}}=-\frac{3}{9}=-\frac{1}{3} \\ \\ y=-3\cdot3^{-1}=-\frac{3}{3^1}=-\frac{3}{3}=-1 \\ \\ y=-3\cdot3^0=-3\cdot1=-3 \\ \\ y=-3\cdot3^1=-3\cdot3=-9 \\ \\ y=-3\cdot3^2=-3\cdot9=-27 \\ \\ y=-3\cdot3^3=-3\cdot27=81\end{cases}[/tex]

So, the table is,

Next, we have to graph all of these points in the coordinate plane. The last one cannot be graphed because y = -81 does not fit in the given coordinate plane. Also, the first two values won't be very accurate because of the scale of the y-axis. The graphed points are,

And finally, to graph the function we join the dots with a line.

Read the proof. Statement Reason 1. given Given: AE1 EC; BD 1 DC 1. AEI EC;BD IDC Prove: AAEC - ABDC A 2. ZAEC is a rt. 2; ZBDC 2. definition of is a rt. 2 perpendicular 3. ZAEC • ZBDC 3. all right angles are congruent 4. ? 4. reflexive property 5. AAEC - ABDC 5. AA similarity theorem. What is the missing statement in step 4? B D O ZACE = ZBCD O ZEAB DBC O ZEAC LEAC O ZCBD ZDBC

Answers

Answer:

  (a)  ∠ACE≅∠BCD

Step-by-step explanation:

You want to know the missing statement in the proof that goes with reason "Reflexive Property."

Proof

You are proving two triangles are similar by showing two corresponding angles are congruent. Corresponding angles in the two triangles are ...

EAC and DBCAEC and BDCACE and BCD

The proof already shows AEC is congruent to BDC in statement 3.

Reflexive property

The reflexive property says an angle is congruent to itself. Looking at the list of corresponding angles, the only angle that corresponds to itself is angle C, which can be named ∠ACE or ∠BCD.

The appropriate choice is ...

  ∠ACE≅∠BCD . . . . Reflexive property

<95141404393>

The missing statement in step 4 should be:ZACE = ZBCD

This statement is missing from the proof and should be included to establish the congruence between the angles in the two triangles.Let's go through the proof step by step and explain each statement and reason.

Given:

AE = EC; BD = DC

AEI = EC; BD = DC

Reason: Given

ZAEC is a right angle; ZBDC is a right angle

Reason: Definition of a right angle. This statement indicates that angle ZAEC and angle ZBDC are both right angles.

ZAEC ≅ ZBDC

Reason: All right angles are congruent. This statement asserts that angle ZAEC and angle ZBDC are congruent (have the same measure) because they are both right angles.

[Missing Statement]

Reason: Reflexive property. This statement is missing in the proof and should be included. The reflexive property states that any angle is congruent to itself. In this case, it implies that angle ZAEC is congruent to angle ZAEC.

AAEC ≅ ABDC

Reason: AA similarity theorem. This statement indicates that triangle AAEC is congruent to triangle ABDC. The AA similarity theorem states that if two pairs of corresponding angles in two triangles are congruent, then the triangles are similar.

So, to complete the proof, the missing statement in step 4 should be:

ZACE = ZBCD

Reason: Reflexive property. This statement establishes that angle ZACE is congruent to angle ZBCD, based on the reflexive property.

Learn more about statement here:

https://brainly.com/question/9048478

#SPJ8

Look for a pattern in the following list. Then use this pattern to predict thenext number. 2, -2, 3, -3, 4, ... *

Answers

Here, we are given the following numbers:

2, -2, 3, -3, 4.........

The pattern here is that a positive integer is followed by its negative value.

We can see that the number after 2 is its negative value -2

The number after 3 is its negative vaule -3

The number after 4 will be its negative which is -4

ANSWER:

-4

multiply and simplify (5x−4√5)(5x+4√5)

Answers

Answer::

[tex]25x^2-80[/tex]

Explanation:

Given the product:

[tex]\left(5x−4\sqrt{5}\right)\left(5x+4\sqrt{5}\right)[/tex]

First, expand the brackets:

[tex]\begin{gathered} =5x\left(5x+4\sqrt{5}\right)−4\sqrt{5}\left(5x+4\sqrt{5}\right) \\ =(5x)^2+20x\sqrt{5}-20x\sqrt{5}-(4\sqrt{5})^2 \\ =(5x)^2-(4\sqrt{5})^2 \end{gathered}[/tex]

We then simplify:

[tex]\begin{gathered} =5^2x^2-4^2\sqrt{5}^2 \\ =25x^2-16(5) \\ =25x^2-80 \end{gathered}[/tex]

The simplified form of the product is:

[tex]25x^2-80[/tex]

Please help me to select the correct image for the representation of the function f(x) = 4 x3x?

Answers

Answer:

Explanation:

Given the below exponential function;

[tex]f(x)=4\cdot3^x[/tex]

To be able to graph the above function, we'll go ahead and choose different values for x and determine the corresponding values of f(x).

When x = 0, we'll have;

[tex]f(0)=4\cdot3^0=4\cdot1=4[/tex]

Looking at all the given four graphs, we can observe that only one of them has a y-interce

A phone company offers two monthly charge plans. In Plan A, the customer pays a monthly fee of $30 and then an additional 9 cents per minute of use. In PlanB, the customer pays a monthly fee of $33.60 and then an additional 8 cents per minute of use.For what amounts of monthly phone use will Plan A cost less than Plan B?Use m for the number of minutes of phone use, and solve your Inequality for m.

Answers

Let m denote the number of minutes.

Plan A:

The customer pays a monthly fee of $30 and then an additional 9 cents per minute of use.

Mathematically,

[tex]30+0.09m[/tex]

Plan B:

The customer pays a monthly fee of $33.60 and then an additional 8 cents per minute of use.

Mathematically,

[tex]33.60+0.08m[/tex]

For what amounts of monthly phone use will Plan A cost less than Plan B?

[tex]30+0.09m<33.60+0.08m[/tex]

Let us solve the above inequality for m

[tex]\begin{gathered} 30+0.09m<33.60+0.08m \\ 0.09m-0.08m<33.60-30 \\ 0.01m<3.60 \\ m<\frac{3.60}{0.01} \\ m<360 \end{gathered}[/tex]

This means that for less than 360 minutes, plan A will cost less than Plan B.

can you help me with the 4th question which is marked b

Answers

we are given the following equation:

[tex]3x+2y=12[/tex]

The slope-intercept form is the following:

[tex]y=mx+b[/tex]

Therefore, we need to solve for "y" in the equation. To do that we will subtract "3x" to both sides:

[tex]\begin{gathered} 3x-3x+2y=12-3x \\ 2y=12-3x \end{gathered}[/tex]

Now we will divide both sides by "2":

[tex]y=\frac{12-3x}{2}[/tex]

Now we will separate the numerator:

[tex]y=\frac{12}{2}-\frac{3}{2}x[/tex]

simplifying:

[tex]y=6-\frac{3}{2}x[/tex]

And thus we get the slope-intercept form.

I need the steps on how to go about this

Answers

Answer:

Explanation:

Consider the line y=2x/3 - 7 Find the equation of the line that is perpendicular to this line and passes through the point (2, 6)Find the equation of the line that is parallel to this line and passes through the point (2, 6)Equation of perpendicular line: Equation of Parallel line:

Answers

The equation of a line in the slope intercept form is expressed as

y = mx + c

where

m = slope

c = y intercept

The given equation is

y = 2x/3 - 7

By comparing both equations,

m = 2/3

If two lines are perpendicular, it means that the slope of one line is equal to the negative reciprocal of the slope of the other line. This means that the slope of the perpendicular line passing through the point (2, 6) is the negative reciprocal of 2/3. It is - 3/2

Thus, m = - 3/2

We would find the y intercept of the perpendicular line by substituting m = - 3/2, x = 2 and y = 6 into the slope intercept equation. We have

6 = - 3/2 * 2 + c

6 = - 3 + c

c = 6 + 3 = 9

By substituting m = - 3/2 and c = 9 into the slope intercept equation, the equation of the perpendicular line is

y = - 3x/2 + 9

Also,

If two lines are parallel, it means that the slope of one line is equal to the slope of the other line. This means that the slope of the parallel line passing through the point (2, 6) is 2/3

Thus, m = 2/3

We would find the y intercept of the perpendicular line by substituting m = 2/3, x = 2 and y = 6 into the slope intercept equation. We have

6 = 2/3 * 2 + c

6 = 4/3 + c

c = 6 - 4/3 = 14/3

By substituting m = 2/3 and c = 14/3 into the slope intercept equation, the equation of the parallel line is

y = 2x/3 + 14/3

Using the slope formula, find the slope of the line through the given points.(-3,-7) and (8,-7)

Answers

the slope of the line is 0

Explanation

The slope of a line is a measure of its steepness of a line , The vertical change between two points is called the rise, and the horizontal change is called the run. The slope equals the rise divided by the run:

[tex]\begin{gathered} slope=\frac{rise}{run}=\frac{change\text{ in y}}{change\text{ in x}}=\frac{y_2-y_1}{x_2-x_1} \\ where \\ P1(x_1,y_1\text{ \rparen and P2\lparen x}_2,y_2)\text{ are 2 points from the line} \end{gathered}[/tex]

so

Step 1

given

[tex]\begin{gathered} P1=(-3,-7) \\ P2=(8,-7) \end{gathered}[/tex]

replace in the formula

[tex]\begin{gathered} slope=\frac{y_{2}-y_{1}}{x_{2}-x_{1}} \\ slope=\frac{-7-(-7)}{8-(-3)}=\frac{-7+7}{11}=\frac{0}{11}=0 \end{gathered}[/tex]

hence, the slope of the line is 0

I hope this helps you

Each person in a group of students was identified by year and asked when he or she preferredtaking classes: in the morning, afternoon, or evening, The results are shown in the table. Findthe probability that the student preferred afternoon classes given he or she is a junior. Roundto the nearest thousandth. When Do You Prefer to Take Classes?

Answers

In this case, we'll have to carry out several steps to find the solution.

Step 01:

Data

Junior

Morning 17

Afternoon 20

Evening 3

Step 02:

Junior

probability afternoon = junior afternoon / total junior afternoon

total junior afternoon = 17 + 20 + 3 = 40

probability afternoon = 20 / (17 + 20 + 3) = 0.5

The answer is:

probability afternoon = 0.5

Ms. Wong wrote a test. Part A had true/false questions, each worth 7 points. Part B had multiple choice questions, each worth 3 points. She made the number of points for Part A equal the number of points for Part B. It was the least number of points for which this was possible.
Answer the following questions.

How many points was each part worth?

How many questions did Part A have?

How many questions did Part B have?

Answers

Considering the least common multiple of 7 and 3, it is found that:

Each part was worth 21 points.Part A had 3 questions.Part B had 7 questions;

How to obtain the measures?

The amount of points of each question in each part are given as follows:

Part A: 7 points.Part B: 3 points.

Both parts have the same number of points, and this amount was the least number of points for which this was possible, hence this amount is the least common multiple of 7 and 3.

Both 3 and 7 are prime numbers, hence the least common multiple of 3 and 7 is given by their multiplication, as follows:

7 x 3 = 21.

Hence each part of the test was worth 21 points.

The number of questions of each part is given by the division of 21 and the worth of each question, hence:

Part A: 21/7 = 3 questions.Part B: 21/3 = 7 questions.

More can be learned about the least common multiple at https://brainly.com/question/10749076

#SPJ1

Write logaa=4x in exponential form and find x to evaluate logaa for any a>0, a≠1.

Answers

Given:

[tex]\log _aa=4x[/tex]

To find the exponential form of the above, all we need to do is to raise the base a to the power of 4x.

That is;

[tex]a^{4x}=a[/tex]

To find the value of x, we need to raise the power of the right - hand side so that we can equate the exponent

That is;

[tex]a^{4x}=a^1[/tex]

4x = 1

Divide both-side by 4

[tex]x=\frac{1}{4}[/tex]

[tex]\text{Log}_aa=4(\frac{1}{4})[/tex][tex]\text{Log}_aa=1[/tex]

calculate the length of side AC​

Answers

Answer:

×=12+5

×=144+25

×=169

×=13

select the expression that will calculate how many eighths are in 2 bars

Answers

Answer:

Explanations:

I am doing an equation trying to figure out a formula for the volume of a box and I am so lost I will include a picture

Answers

The volume of any rectangular box is expressed as:

[tex]\text{Volume}=\text{length}\times\text{breadth}\times height[/tex]

Now, for the box that will be formed from the figure shown in the question, we will have:

length = 37 - 2x

breadth = 37 - 2x

height = x

Thus, we have that:

[tex]\begin{gathered} \text{Volume}=\text{length}\times\text{breadth}\times height \\ \Rightarrow\text{Volume}=(37-2x)\times(37-2x)\times x \end{gathered}[/tex]

We now simplify the above as:

[tex]\begin{gathered} \text{Volume}=(37-2x)\times(37-2x)\times x \\ \Rightarrow\text{Volume}=(1369-148x+4x^2)\times x \\ \Rightarrow\text{Volume}=1369x-148x^2+4x^3 \\ \Rightarrow\text{ V(x)}=1369x-148x^2+4x^3 \end{gathered}[/tex]

Now that we have obtained the expression for the volume of the box, we now have to find the value of x that maximizes it.

This is done as follows:

Method

- Differentiate the function V(x) with respect to x, and equate to zero as follows:

[tex]\begin{gathered} \Rightarrow V^1\text{(x)}=1369-296x^{}+12x^2 \\ \text{Equating to zero:} \\ 1369-296x^{}+12x^2=0 \\ \text{The roots of the equation are:} \\ \Rightarrow x=6.167\text{ and x = }18.5 \end{gathered}[/tex]

Now we have to find the second derivative of V(x) in order to confirm which value of x makes the function V(x) a maximum

Thus:

[tex]\begin{gathered} \Rightarrow V^{11}\text{(x)}=-296^{}+24x^{} \\ \text{when x = 6.167} \\ \Rightarrow V^{11}\text{(6.167)}=-296^{}+24(6.167)=-296+148.008=-148 \\ \text{when x = }18.5 \\ \Rightarrow V^{11}\text{(18.5)}=-296^{}+24(18.5)=-296+444=148 \end{gathered}[/tex]

Now since the second derivative is a negative number when x = 6.167, we now know for sure that it is that value of x that maximizes the function V(x), and not x = 18.5.

Thus, we can conclude that the value of x that maximizes the volume of the box is:

x = 6.17 inches (to 2 decimal places)

If we had been asked to find the value of x that minimizes the volume, the answer will have been x = 18.5, because this value of x made the second derivative of V(x) positive.

Now, the maximum volume of the box is obtained by simply substituting the value of x that maximizes the function into the original expression for V(x), as follows:

[tex]\begin{gathered} V(x)=1369x-148x^2+4x^3 \\ \text{when x= 6.167} \\ \Rightarrow\text{ V(6.167)}=1369(6.167)-148(6.167)^2+4(6.167)^3 \\ \Rightarrow\text{ V(6.167)}=8442.623-5628.720+938.171 \\ \Rightarrow\text{ V(6.167)}=3752.074in^3 \\ \Rightarrow\text{ V(6.167)}=3752.07in^3\text{ (to 2 decimal places)} \end{gathered}[/tex]

Solve the equation for y in terms of x. In other words, algebraicallyrearrange the equation so that the y variable is by itself one side of theequation. Type your answer in the form y = mx + b. If you have a valuethat is not an integer then type it rounded to the nearest hundredth. Donot put spaces between your characters.4x + 2y = 8y = ?

Answers

We can determine an expression of y in terms of x by isolating y on one side of the equation, we can do this by means of some algebraic operations to get:

4x + 2y = 8

1. Subtract 4x from both sides of the equation:

4x - 4x + 2y = 8 - 4x

0 + 2y = 8 - 4x

2y = 8 - 4x

2. Divide both sides by 2

2y/2 = (8 - 4x)/2

y = 4 - 2x

y = -2x + 4

Then, the equation of y in terms of x is y=-2x+4

32. What is the rate of change of y with the respect to x for 24x - 4y = 50

Answers

The equation for the graph is given as

[tex]24x-4y=50[/tex]

Let us rearrange the equation into its Slope-Intercept form given as

[tex]y=mx+c[/tex]

Where

m = rate of change

c = y-intercept

Therefore, we will have

[tex]-4y=-24x+50[/tex]

Divide all terms by -4 to make y a standalone variable:

[tex]\begin{gathered} \frac{-4y}{-4}=\frac{-24x}{-4}+\frac{50}{(-4)} \\ y=6x-\frac{25}{2} \end{gathered}[/tex]

Comparing with the Slope-Intercept equation, the rate of change is given as 6.

AC⌢ =84 ∘ , find m∠ADC.

Answers

The measure of minor arc is 84 degree

The expression for the an angle inscribed in a circle, then the measurement of the angle is equal to the half of the measure of its intercepted arc.

[tex]\text{Angle}=\frac{1}{2}m(arc)[/tex]

here we have, arc length = 84 degree

[tex]\begin{gathered} m\angle ADC=\frac{1}{2}(mAC) \\ m\angle ADC=\frac{1}{2}\times84 \\ m\angle ADC=42^o \end{gathered}[/tex]

Angle = 42 degree

The number line below represents which combined inequality? xs-6 orx 25 xs -6 and x 2 5 X2 -6 and x s 5 x2-6 or x s 5

Answers

Answer

Option C is correct.

x ≥ -6 and x ≤ 5

Explanation

In graphing inequality equations, the first thing to note is that whenever the equation to be graphed has (< or >), the circle at the beginning of the arrow is usually unshaded.

But whenever the inequality has either (≤ or ≥), the circle at the beginning of the arrow will be shaded.

Since the beginning of the blue mark is a shaded circle, the inequality is (≤ or ≥).

And considering that the region of the answer is between -6 and 5, it is evident that x is greater than or equal to -6 and less than or equal to 5. In mathematical terms,

x ≥ -6 and x ≤ 5

-6 ≤ x ≤ 5

Hope this Helps!!!

The cubic function f(x) = z3 - 6x2 + 11x - 6 has a root at z = 3. a What are the other roots of the function?O r = 3, x = 2O r =-3, x = -2O x = -1, x = -2O x = 1, r = 2

Answers

Given the function f(x) as follows:

[tex]f\mleft(x\mright)=x^3-6x^2+11x-6[/tex]

The function has a root at x = 3

We will use the synthetic division to find the other roots:

We will divide the coefficients by 3

As follows:

So, the given function will be written as follows:

[tex]f(x)=(x-3)(x^2-3x+2)[/tex]

Factor the term of the quadratic function

[tex]f(x)=(x-3)(x-2)(x-1)[/tex]

So, there are three zeros x = 1, 2, 3

So, the answer will be option 4) x = 1, x = 2

A- what is R(300) interpret this result B- what is the revenue from the sale of 2,000 hats write in functional notation3 part question

Answers

Okay, here we have this:

Considering the provided information, and the given function we are going to calculate R(300) and then we will interpret the result, so we obtain the following:

[tex]\begin{gathered} R(x)=17x \\ R(300)=17\cdot300 \\ R(300)=5100 \end{gathered}[/tex]

Considering that x corresponds to the number of hats sold, then it means that if 300 hats are sold, the total revenue will be equal to $5100.

peter is paid k500.00 for the work in 18 hours. how much would he be paid if he had worked six hours

Answers

Given:

500 Kina for 18 hours of work

To determine the amount of payment if he had worked for 6 hours, we use ratio.

So,we let x be the amount of payment for 6 hours of work:

[tex]\begin{gathered} \frac{500\text{ Kina}}{18\text{ hours}}=\frac{x}{6\text{ hours}} \\ \text{Simplify and rearrange} \\ x=\frac{500(6)}{18} \\ \text{Calculate} \\ x=166.67\text{ Kina} \end{gathered}[/tex]

Therefore, he would be paid 166.67 Kina if he had worked for six hours.

Joshua has $1.20 worth of nickels and dimes. He has 6 more nickels than dimes.
Graphically solve a system of equations in order to determine the number of nickels,
x, and the number of dimes, y, that Joshua has.
40
38
36
34
32
30
28
26
24
22
20
18
16
14
12
10
8
0
Click twice to plot each line. Click a line to delete it.
2 4 6 8 10 12 14 16 18 20 22 24 26 28 30 32 34 36 38 40

Answers

Answer:

12 nickels, 6 dimes

Step-by-step explanation:

0.05x + 0.1y = 1.20
x - 6 = y

0.05x + 0.1(x-6) = 1.20
0.05x + 0.1x - 0.6 = 1.20
0.15x = 1.80
x = 12

(12) - 6 = y
y = 6

Now graph y = x - 6 and y = (-1/2)x + 12

If you don't know how to graph the functions, then go to khan academy for help.

A triangle has sides with lengths of 12 yard, 13 yards, and 15 yards. Which numbers are representing the legs of a triangle?

Answers

ANSWER

None of these sides represent the legs.

EXPLANATION

The legs of a triangle are the sides that form the right angle in a right triangle. The legs are always the shortest sides, while the hypotenuse is the longest side.

In this case, the hypotenuse would be 15 yards, while the legs would be 12 yards and 13 yards. For this to be a right triangle, the Pythagorean Theorem must be satisfied,

[tex]\begin{gathered} 12^2+13^2=15^2 \\ 144+169=15^2 \\ 313=225\to false \end{gathered}[/tex]

These side lengths do not satisfy the Pythagorean Theorem and, therefore, this is not a right triangle. If it is not a right triangle, then it does not have legs. Hence, none of these represent the legs.

Which exponential expressions are equivalent to the one below? Check allthat apply.(3.7) 10A. 310 + 710B. (3:7)10O .C. 2110O d. 310.710

Answers

Given the exponential expression:

[tex](3\cdot7)^{10}[/tex]

The equivalent expressions are:

[tex]\begin{gathered} (3\cdot7)^{10}=3^{10}\cdot7^{10} \\ (3\cdot7)^{10}=21^{10} \end{gathered}[/tex]

So, the answer will be options C, D

Other Questions
you have been offered admission into senior high school but you have not been able to report at the school.write a letter to the head of the school telling him about your problems and asking him to give a you few more days to report There are 2 main types of _______. One is a _______, where electrons are shared between atoms. The other is an _______, where one atom loses an electron to become a _______ charged ion, and the other atom gains the elctron to become a _________ charged ion. Which equation can be solved by using this system of equations? startlayout enlarged left-brace 1st row y = 3 x cubed minus 7 x squared 5 2nd row y = 7 x superscript 4 baseline 2 x endlayout One number is five more than three times another. If their sum is increased by one, the result is twenty-six. Find the numbers.The smaller of the numbers is ? and the larger is ? . I get 15 percent discount at a store if I find a I like for 40 how much will I have to pay for it Suppose f(x)= 2+4x^2.Simplify as much as possible: f(1)/f(2)= ______ You invested $5000 between two accounts paying 7% and 8% annual interest. If the total interest earned for the year was $380, how much was invested at each rate? Mrs. Peck is making school supply baskets. She purchased 27 composition booksand 9 packs of map pencils. Which shows the ratio of packs of map pencils tocomposition books. Consider the functions f(x)=4-X^2 and g(x)=3x+5.Find the value of f(g)g-2))). c-884= -853solve for c why would Mansa and musa send the sultan a gift but then refuse the offer to meet him? A. Find the domain of f(x). Write your answer in interval notation.B. Find the range of f(x). Write your answer in interval notation.C. Find the following:i. f(0)ii. f(-2)iii. f(8)iv. f(3)V. fl-1)D. Find all x's (approximately) such that f(x)=1. True or false. A 3. 0-mole sample of co2 gas effused through a pinhole in 18. 0 s. It will take 1. 92 s for the same amount of h2 to effuse under the same conditions. How many moles are in 28.87 grams of Ec3H305?Ec has a molar mass of 31.79 grams/mol.(Hint: You need to determine the molar mass of Ec3H305 to solve this problem). suppose you want to subtract: -4-(-2) Joanna is wrapping a present in the box shown.find the amount of wrapping paper in square inches that Joanna needs In the reaction H + I --> 2HI, what is the oxidation number of the hydrogen in the H reactant? (2 pts)*2 points-1012 Which of the following is not a life role category?O Risk TakerO WorkerO Family MemberO Learner If the probability of an event is what is the probability of the event not happening?20/69Write your answer as a simplified fraction. select the point(s) of the x intercept of the function shown below